M

Chiudi

Teoremi di De L’Hôpital: esercizi misti sui limiti

Teorema di De L'Hôpital

Home » Teoremi di De L’Hôpital: esercizi misti sui limiti

Teoremi di De L’Hôpital: esercizi misti sui limiti

I teoremi di De L’Hôpital sono un importante e utile strumento dell’Analisi Matematica, sia teorico, nella dimostrazione di altri teoremi, sia pratico, per la risoluzione di forme indeterminate altrimenti difficilmente trattabili.

In questo articolo proponiamo 21 esercizi completamente risolti mediante l’utilizzo dei teoremi, di diversa difficoltà, che integrano inoltre diverse tecniche e mostrano come la loro combinazione possa diventare un efficace mezzo per giungere alla soluzione di un problema.

La raccolta è dunque indicata sia per gli studenti dei corsi di Analisi Matematica 1 che vogliono allenare la loro preparazione in vista dell’esame, sia a chi semplicemente desidera approfondire le potenzialità di questo interessante strumento.
Auguriamo quindi a tutti una buona lettura!

Consigliamo le seguenti raccolte di esercizi su argomenti correlati:

La teoria di riferimento può essere reperita ai seguenti link, estratti dalla lista completa situata alla fine dell’articolo:

Autori e Revisori

Leggi...

Autore: Giulio Pecorella.

Revisori: Luigi De Masi, Valerio Brunetti, Matteo Talluri.


 

Notazioni

Leggi...

\mathbb{Z}

\mathbb{R}

(a,b)

[a,b]

Insieme dei numeri interi;

Insieme dei numeri reali;

Intervallo aperto di estremi a,b;

Intervallo chiuso di estremi a,b

    \[\,\]

    \[\quad\]


 

Introduzione

Leggi...

In questa dispensa vengono usati i teoremi di de L’Hôpital nello studio di alcune forme indeterminate del tipo \left[\frac{0}{0}\right] e \left[\frac{\infty}{\infty}\right]. Vengono inoltre mostrati alcuni casi in cui i teoremi non sono applicabili, e si mostra come affrontare questi casi. La dispensa è così organizzata:  

  1. La sezione 1 è dedicata ad un breve richiamo sui teoremi.
  2. La sezione 2 riporta una serie di esercizi di difficoltà crescente con le relative risoluzioni.

Alla fine della dispensa sono presenti alcuni riferimenti bibliografici sull’argomento.

 

Richiami di teoria

Leggi...

In questa sezione riportiamo due versioni del teorema di de L’Hôpital, le altre sono ottenibili da queste tramite facili manipolazioni algebriche e cambi di variabile. Per una trattazione completa con annesse dimostrazioni si rimanda il lettore a [3].

Teorema 1 ( de L’Hôpital, caso \left[\frac{0}{0}\right] ). Siano a,b\in \mathbb{R}, a<b, x_0\in[a,b] e siano f,g\colon (a,b)\setminus \{x_0\} \to \mathbb{R} due funzioni derivabili in (a,b)\setminus \{x_0\} tali che:

 

  1. \displaystyle 	\lim\limits_{x\to x_0}f(x)=\lim\limits_{x\to x_0}g(x)=0;
  2.  

  3. g'(x)\neq 0\;\; \forall x \in (a,b)\setminus \{x_0\};
  4.  

  5. esiste

    (1)   \begin{equation*} 	\lim\limits_{x\to x_0}\frac{f'(x)}{g'(x)}=\ell\in \mathbb{R}\cup\{\pm \infty\}. \end{equation*}

Allora

(2)   \begin{equation*} 	\lim\limits_{x\to x_0}\frac{f(x)}{g(x)}=\ell. \end{equation*}

 

Facciamo alcune osservazioni:

  • Il teorema 1 si generalizza al caso in cui x_0=\pm\infty. Infatti, supponiamo che f,g \colon (b,+\infty) \to \mathbb{R} soddisfino le ipotesi del teorema 1 con +\infty al posto di x_0. A meno di restringere il dominio di f e g, possiamo assumere che b>0. Consideriamo le funzioni \tilde{f},\tilde{g} \colon \left (0, \frac{1}{b} \right) \to \mathbb{R} definite da

     

    (3)   \begin{equation*} 		\tilde{f}(x) = f \left ( \frac{1}{x} \right ), 		\quad 		\tilde{g}(x) = g \left ( \frac{1}{x} \right ), 		\qquad 		\forall x \in \left (0, \frac{1}{b} \right). 	\end{equation*}

    Si verifica facilmente che \tilde{f},\tilde{g} soddisfano le ipotesi del teorema 1 con x_0=0. Infatti, ad esempio si ha

    (4)   \begin{equation*} 		\lim_{t \to 0^+} 		\frac{\tilde{f}'(t)}{\tilde{g}'(t)} 		= 		\lim_{t \to 0^+} 		\frac{-f'\left ( \frac{1}{t} \right ) \frac{1}{t^2}}{-g'\left ( \frac{1}{t} \right )\frac{1}{t^2}} 		= 		\lim_{t \to 0^+} 		\frac{f'\left ( \frac{1}{t} \right ) }{g'\left ( \frac{1}{t} \right )} 		= 		\lim_{x \to +\infty} \frac{f'(x)}{g'(x)} 		=\ell, 	\end{equation*}

    dove nella penultima uguaglianza si è usato il cambiamento di variabile t=\frac{1}{x}. Quindi, applicando il teorema 1 alle funzioni \tilde{f},\tilde{g} in 0 si ottiene la conclusione anche su f,g, in quanto, di nuovo per il cambiamento di variabili t=\frac{1}{x}, vale

    (5)   \begin{equation*} 		\ell 		= 		\lim_{t \to 0} = \frac{\tilde{f}(t)}{\tilde{g}(t)} 		= 		\lim_{x \to +\infty} \frac{f(x)}{g(x)}. 	\end{equation*}

  • Nel caso in cui il limite del rapporto delle derivate ci dia ancora una forma indeterminata, possiamo applicare il teorema alle derivate: ipotizziamo infatti che

    (6)   \begin{equation*} 		\lim\limits_{x\to x_0} \frac{f'(x)}{g'(x)}, 	\end{equation*}

    sia ancora una forma indeterminata del tipo \left[\frac{0}{0}\right]. Se f' e g' verificano le prime due ipotesi del teorema 1, possiamo provare nuovamente ad applicarlo e calcolare

    (7)   \begin{equation*} 		\lim\limits_{x\to x_0} \frac{f''(x)}{g''(x)}. 	\end{equation*}

    Se tale limite non è una forma indeterminata, allora per il teorema 1 coinciderà col valore del limite (6), che sempre per il teorema 1 coincide con il limite del rapporto f/g. Se invece il limite (7) è ancora una forma indeterminata, si può ancora provare ad applicare il teorema, passando alle derivate terze e via dicendo.

    Enunciamo quindi il caso in cui la forma indeterminata sia del tipo \left[\frac{\infty}{\infty}\right].

    Teorema 2 (de L’Hôpital, caso \left[\frac{\infty}{\infty}\right]). Siano a,b\in \mathbb{R},a<b, x_0\in[a,b] e siano f,g\colon (a,b)\setminus \{x_0\} \to \mathbb{R} due funzioni derivabili in (a,b)\setminus \{x_0\} tali che:

     

    1. \displaystyle\lim\limits_{x\to x_0}f(x)=\pm\infty,\lim\limits_{x\to x_0}g(x)=\pm\infty;
    2.  

    3. g'(x)\neq 0\;\; \forall x \in (a,b)\setminus \{x_0\};
    4.  

    5. esiste

      (8)   \begin{equation*}	 	\lim\limits_{x\to x_0}\frac{f'(x)}{g'(x)}=\ell\in \mathbb{R}\cup\{\pm \infty\}. \end{equation*}

      1. Allora

        (9)   \begin{equation*} 	\lim\limits_{x\to x_0}\frac{f(x)}{g(x)}=\ell. \end{equation*}

     

    Valgono le stesse osservazioni fatte nel caso della forma indeterminata \left[\frac{0}{0}\right].


 

Testi degli esercizi

Esercizio 1   (\bigstar\bigstar\largewhitestar\largewhitestar\largewhitestar). Determinare per quali valori dei parametri a \neq 0 e b>0 vale la formula

(10)   \begin{equation*} 	\lim_{x \rightarrow 0^+}\frac{a\left(1-\cos (x)\right)-2x^b}{x^2}=0. \end{equation*}

Svolgimento.

Ci troviamo di fronte ad una forma indeterminata del tipo \left[\frac{0}{0}\right]. Consideriamo quindi le funzioni f,g\colon \mathbb{R} \to \mathbb{R} definite da

(11)   \begin{equation*} 	f(x)=a(1-\cos(x))-2x^b,\qquad g(x)=x^2\qquad \forall x \in \mathbb{R}. \end{equation*}

Queste funzioni sono derivabili con derivata

(12)   \begin{equation*} 	f'(x)=a\sin(x)-2bx^{b-1},\qquad g'(x)=2x\qquad \forall x \in \mathbb{R}. \end{equation*}

Se calcoliamo il limite del rapporto delle derivate, otteniamo che se b>2

(13)   \begin{equation*} 	\lim\limits_{x\to 0^+}\frac{f'(x)}{g'(x)}=\lim\limits_{x\to 0^+}\frac{a\sin(x)-2bx^{b-1}}{2x}=\frac{a}{2}, \end{equation*}

dove abbiamo utilizzato il limite notevole

(14)   \begin{equation*} 	\lim\limits_{t \to 0^+}\frac{\sin(t)}{t}=1. \end{equation*}

In particolare osserviamo che non esiste alcun valore di a\neq 0 per cui il limite (13) sia pari a 0. Per lo stesso motivo si ha

(15)   \begin{equation*} 	\lim\limits_{x\to 0}\frac{a\sin(x)-4x^{b-1}}{2x}=-\infty, \end{equation*}

se b<2, invece se b=2 si ha

(16)   \begin{equation*} 	\lim\limits_{x\to 0}\frac{a\sin(x)-2bx^{b-1}}{2x}=\frac{a}{2}-b=\frac{a}{2}-2. \end{equation*}

Quindi, se a=4 e b=2, per il teorema 1 si ha

(17)   \begin{equation*} 	\lim_{x \rightarrow 0^+}\frac{a\left(1-\cos (x)\right)-2x^b}{x^2}=0	. \end{equation*}

 

Esercizio 2   (\bigstar\largewhitestar\largewhitestar\largewhitestar\largewhitestar). Si calcoli, se esiste, il seguente limite, applicando il teorema di de L’Hôpital:

(18)   \begin{equation*} 	\lim\limits_{x\to 1}\frac{\log(2x^2-1)}{x^3-1}. \end{equation*}

Svolgimento.

Ci troviamo di fronte ad una forma indeterminata del tipo \left[\frac{0}{0}\right]. Volendo analizzare il limite per x \to 1, consideriamo le funzioni f,g\colon \left(\frac{1}{\sqrt{2}},+\infty\right)\to \mathbb{R} definite da

(19)   \begin{equation*} 	f(x)=\log(2x^2-1),\qquad g(x)=x^3-1\qquad \forall x \in \left(\frac{1}{\sqrt{2}},+\infty\right). \end{equation*}

Notiamo che tali funzioni sono derivabili con derivate

(20)   \begin{equation*} 	f'(x)=\frac{4x}{2x^2-1},\qquad g'(x)=3x^2\qquad \forall x 	\in\left(\frac{1}{\sqrt{2}},+\infty\right). \end{equation*}

Osserviamo che g'(x)\neq 0 per ogni x \in \left(\frac{1}{\sqrt{2}},+\infty\right). Un calcolo diretto ci dice che il limite del rapporto delle derivate è

(21)   \begin{equation*} 	\lim\limits_{x\to 1}\frac{f'(x)}{g'(x)}=	\lim\limits_{x\to 1}\frac{4x}{2x^2-1}\frac{1}{3x^2}=\frac{4}{3}. \end{equation*}

Per il teorema 1 tale valore è quindi pari al valore del limite iniziale. Quindi si ha che

(22)   \begin{equation*} 	\lim\limits_{x\to 1}\frac{\ln(2x^2-1)}{x^3-1}=\frac{4}{3}. \end{equation*}

Osserviamo che possiamo ottenere lo stesso risultato senza applicare il teorema 1, ma applicando il limite notevole del logaritmo. Infatti, con il cambio di variabile y=x-1 il limite diventa

(23)   \begin{equation*} 	\lim\limits_{y\to 0} \frac{\log(1+2y^2+4y)}{y^3+3y^2+3y}=\lim\limits_{y\to 0} \frac{\log(1+2y^2+4y)}{2y^2+4y}\frac{2y^2+4y}{y^3+3y^2+3y}=\frac{4}{3}, \end{equation*}

dove abbiamo applicato il limite notevole del logaritmo

(24)   \begin{equation*} 	\lim\limits_{t\to 0}\frac{\log(1+t)}{t}=1. \end{equation*}

 

Esercizio 3   (\bigstar\largewhitestar\largewhitestar\largewhitestar\largewhitestar). Si calcoli, se esiste, il seguente limite, applicando il teorema di de L’Hôpital:

(25)   \begin{equation*} 	\lim\limits_{x\to \frac{\pi}{2}^-}\frac{\mathrm{e}^{\tan{(x)}}}{\tan{(x)}}. \end{equation*}

Svolgimento.

Si ha una forma indeterminata del tipo \left[\frac{\infty}{\infty}\right]. Nell’analizzare il limite per x\to \frac{\pi}{2}^-, possiamo studiare un opportuno intorno sinistro di \frac{\pi}{2} in cui la tangente è ben definita. Consideriamo quindi le funzioni f,g\colon\left(0,\frac{\pi}{2}\right)\to \mathbb{R} definite da

(26)   \begin{equation*} 	f(x)=\mathrm{e}^{\tan (x)},\qquad g(x)=\tan (x) \qquad \forall x \in \left(0,\frac{\pi}{2}\right). \end{equation*}

Queste funzioni sono derivabili con derivata

(27)   \begin{equation*} 	f'(x)=\frac{\mathrm{e}^{\tan{(x)}}}{\cos^2(x)}\qquad g'(x)=\frac{1}{\cos^2(x)}\qquad \forall x \in \left(0,\frac{\pi}{2}\right). \end{equation*}

Osserviamo che per ogni x\in \left(0,\frac{\pi}{2}\right) si ha g'(x)\neq0. Il limite del rapporto delle derivate ci dà

(28)   \begin{equation*} 	\lim\limits_{x\to \frac{\pi}{2}^-}\frac{f'(x)}{g'(x)}=	\lim\limits_{x\to \frac{\pi}{2}^-} \frac{\mathrm{e}^{\tan{(x)}}}{\cos^2(x)} \cos^2(x)=+\infty. \end{equation*}

Per il teorema 1 si ha quindi

(29)   \begin{equation*} 	\lim\limits_{x\to \frac{\pi}{2}^-}\frac{\mathrm{e}^{\tan{(x)}}}{\tan{(x)}}=+\infty. \end{equation*}

Questo risultato si può ottenere anche applicando la gerarchia degli infiniti: infatti con il cambio di variabile y=\tan (x) il limite precedente è uguale a

(30)   \begin{equation*} 	\lim\limits_{y\to +\infty} \frac{\mathrm{e}^y}{y}=+\infty, \end{equation*}

in quanto l’esponenziale è un infinito di ordine superiore a qualsiasi polinomio, per y\to +\infty.

 

Esercizio 4   (\bigstar\largewhitestar\largewhitestar\largewhitestar\largewhitestar). Si calcoli, se esiste, il seguente limite, applicando il teorema di de L’Hôpital:

(31)   \begin{equation*} 	\lim\limits_{x\to +\infty}\frac{\mathrm{e}^x+5x}{x^2-3x}. \end{equation*}

Svolgimento.

Il limite presenta una forma indeterminata del tipo \left[\frac{\infty}{\infty}\right]. Consideriamo quindi le funzioni f,g\colon\mathbb{R}\to \mathbb{R} definite da

(32)   \begin{equation*} 	f(x)=\mathrm{e}^x+5x,\qquad g(x)=x^2-3x \qquad x \in \mathbb{R}, \end{equation*}

derivabili con derivata

(33)   \begin{equation*} 	f'(x)=\mathrm{e}^x+5,\qquad g'(x)=2x-3 \qquad x \in \mathbb{R}, \end{equation*}

ed inoltre osserviamo che g'(x) si annula solo in x=\frac{3}{2}. Calcolando il limite del rapporto delle derivate si ha ancora una forma indeterminata del tipo \left[\frac{\infty}{\infty}\right]. Seguendo l’osservazione fatta nei richiami di teoria, proviamo ad applicare il teorema 1 alle derivate f' e g', che osserviamo essere derivabili (ad essere precisi f e g sono derivabili infinite volte). Le derivate seconde sono definite dalle leggi

(34)   \begin{equation*} 	f''(x)=\mathrm{e}^x,\qquad g''(x)=2\qquad \forall x \in \mathbb{R}. \end{equation*}

Il limite del rapporto delle derivate seconde vale

(35)   \begin{equation*} 	\lim\limits_{x\to +\infty}\frac{f''(x)}{g''(x)}=\lim\limits_{x\to +\infty}\frac{e^x}{2}=+\infty. \end{equation*}

Per il teorema 2, visto che tale limite esiste questo è pari al limite del rapporto delle derivate prime, che quindi sempre per il teorema 2 è uguale al valore del limite iniziale. In conclusione si ha

(36)   \begin{equation*} 	\lim\limits_{x\to +\infty}\frac{\mathrm{e}^{x}+5x}{x^2-3x}=+\infty. \end{equation*}

Osserviamo infine che questo risultato si può anche ottenere applicando la gerarchia degli infiniti.

 

Esercizio 5   (\bigstar\largewhitestar\largewhitestar\largewhitestar\largewhitestar). Si calcoli, se esiste, il seguente limite, applicando il teorema di de L’Hôpital:

(37)   \begin{equation*} 	\lim\limits_{x\to 2}\frac{\sqrt{x}-\sqrt{2}}{x-2}. \end{equation*}

Svolgimento.

Si ha una forma indeterminata del tipo \left[\frac{0}{0}\right]. Per applicare il teorema 1, consideriamo innanzitutto le funzioni f,g\colon (0,+\infty)\to \mathbb{R} definite dalle leggi

(38)   \begin{equation*}         f(x)=\sqrt{x}-\sqrt{2},\qquad g(x)=x-2 \qquad \forall x \in (0,+\infty). \end{equation*}

Queste funzioni sono derivabili, e le leggi delle derivate sono

(39)   \begin{equation*} 	f'(x)=\frac{1}{2\sqrt{x}},\qquad g'(x)=1 \qquad \forall x \in (0,+\infty). \end{equation*}

Siamo quindi nelle ipotesi del teorema 1, una volta verificato che il limite del rapporto delle derivate esista. Questo limite è pari a

(40)   \begin{equation*} 	\lim\limits_{x\to 2} \frac{f'(x)}{g'(x)}=\lim\limits_{x\to 2} \frac{1}{2\sqrt{x}}=\frac{1}{2\sqrt{2}}. \end{equation*}

Per il teorema 1 si ha quindi

(41)   \begin{equation*} 	\lim\limits_{x\to 2}\frac{\sqrt{x}-\sqrt{2}}{x-2}=\frac{1}{2\sqrt{2}}. \end{equation*}

Osserviamo che avremmo potuto trattare il limite precedente con un’altra strategia. Ricordando la nota proprietà (a^2-b^2)=(a+b)(a-b), possiamo manipolare il limite precedente moltiplicando numeratore e denominare per \sqrt{x}+\sqrt{2}, ottenendo

(42)   \begin{equation*} 	\lim\limits_{x\to 2}\frac{\sqrt{x}-\sqrt{2}}{x-2}=  \lim\limits_{x\to 2}\frac{1}{x-2}\frac{x-2}{\sqrt{x}+\sqrt{2}}=\frac{1}{2\sqrt{2}}, \end{equation*}

coerentemente con il risultato ottenuto in (41).

 

Esercizio 6   (\bigstar\largewhitestar\largewhitestar\largewhitestar\largewhitestar). Si calcoli, se esiste, il seguente limite, applicando il teorema di de L’Hôpital:

(43)   \begin{equation*} 	\lim\limits_{x\to 0}\frac{x^2\sin(x^{-4})}{x}. \end{equation*}

Svolgimento.

È possibile calcolare il limite direttamente, in quanto si può semplificare un fattore x tra numeratore e denominatore, ottenendo

(44)   \begin{equation*} 	\lim_{x \to 0} \dfrac{x^2 \sin(x^{-4})}{x}=\lim_{x \to 0} x \sin(x^{-4})= 0, \end{equation*}

dove l’ultima uguaglianza si ottiene applicando il teorema del confronto, da cui segue che il prodotto di una funzione infinitesima per una funzione limitata è infinitesima: infatti, il fattore x ha limite 0 per x \to 0, mentre il fattore \sin(x^{-4}) è limitato poiché assume valori compresi tra -1 e 1.

Osserviamo che le ipotesi del teorema 1 non sono verificare dalle funzioni al numeratore e al denominatore f,g \colon \mathbb{R} \setminus \{0\} \to \mathbb{R} definite da

(45)   \begin{equation*} 	f(x)= x^2 \sin(x^{-4}), \quad g(x) = x \qquad\forall x \in \mathbb{R} \setminus \{0\}. \end{equation*}

Esse sono entrambe derivabili, con derivate pari a

(46)   \begin{equation*} 	f'(x)= 2x\sin(x^{4}) - 4x^{-3} \cos(x^{-4}),\quad g'(x)=1 \qquad\forall x \in \mathbb{R} \setminus \{0\}, \end{equation*}

ma il limite del rapporto delle derivate

(47)   \begin{equation*} 	\lim_{x \to 0} \frac{f'(x)}{g'(x)}=\lim_{x \to 0} \big( 2x\sin(x^{4}) - 4x^{-3} \cos(x^{-4}) \big), \end{equation*}

non esiste. Infatti, il termine 2x\sin(x^{4}) è infinitesimo in quanto prodotto di una funzione infinitesima per una limitata; d’altra parte, il termine 4x^{-3} \cos(x^{-4}) non ha limite: il fattore 4x^{-3} è divergente, ma il fattore \cos(x^{-4}) assume infinite volte sia il valore 1 che -1 in ogni intorno di 0 e dunque il prodotto non ha limite. L’ipotesi 3 del teorema 1 non è quindi verificata.

 

Esercizio 7   (\bigstar\bigstar\largewhitestar\largewhitestar\largewhitestar). Si calcoli, se esiste, il seguente limite, applicando il teorema di de L’Hôpital:

(48)   \begin{equation*} 	\lim\limits_{x\to \pi}\frac{\log(x-\pi)}{\cot(2x)}. \end{equation*}

Svolgimento.

Nell’analizzare tale limite, osserviamo dapprima che esso ha senso in un intorno destro di \pi (ovvero per x\to \pi^+), affinchè abbia senso il logaritmo al numeratore. Ricordiamo poi che la cotangente è definita come reciproco della tangente (ovvero rapporto di coseno su seno), e quindi si ha

(49)   \begin{equation*} 	\lim\limits_{x\to \pi^+}\cot(2x)=+\infty. \end{equation*}

Quindi ci troviamo di fronte ad una forma indeterminata del tipo \left[\frac{\infty}{\infty}\right]. Nel voler applicare il teorema 2, consideriamo le funzioni f,g\colon\left(\pi,\frac{3}{2}\pi \right)\to \mathbb{R} definite da

(50)   \begin{equation*} 	f(x)=\log(x-\pi),\qquad g(x)=\cot(2x)\qquad \forall x \in \left(\pi,\frac{3}{2}\pi\right). \end{equation*}

Esse sono derivabili con derivate

(51)   \begin{equation*} 	f'(x)=\frac{1}{x-\pi},\qquad 	g'(x)=-\frac{2}{\sin^2(2x)}\qquad\forall x \in (\pi,2\pi). \end{equation*}

Per applicare il teorema 2, una volta osservato che g' non è mai nulla, non ci resta che studiare il limite del rapporto delle derivate

(52)   \begin{equation*} 	\lim\limits_{x\to \pi} \frac{f'(x)}{g'(x)}=\lim\limits_{x\to \pi^+} \frac{1}{x-\pi^+} \frac{(-\sin^2(2x))}{2}. \end{equation*}

Purtroppo ci troviamo di fronte ad una forma indeterminata del tipo \left[\frac{0}{0}\right], per cui dobbiamo studiare in maniera più approfondita quest’ultimo limite. Operiamo il seguente cambio di variabile: y=x-\pi; sfruttando la periodicità del seno otteniamo

(53)   \begin{equation*} 	\lim\limits_{x\to \pi^+} \frac{1}{x-\pi} \frac{(-\sin^2(2x))}{2}=\lim\limits_{y\to 0^+}\frac{-\sin^2(2y)}{2y}=\lim\limits_{y\to 0^+}\frac{-\sin^2(2y)}{(2y)^2}2y=0, \end{equation*}

dove abbiamo sfruttato il limite notevole

(54)   \begin{equation*} 	\lim\limits_{t\to 0}\frac{\sin (t)}{t}=1. \end{equation*}

Possiamo quindi applicare il teorema 1, che ci permette di concludere che

(55)   \begin{equation*} 	\lim\limits_{x\to \pi^+}\frac{\log(x-\pi)}{\cot(2x)}=0. \end{equation*}

 

Esercizio 8   (\bigstar\bigstar\largewhitestar\largewhitestar\largewhitestar). Si calcoli, se esiste, il seguente limite, applicando il teorema di de L’Hôpital:

(56)   \begin{equation*} 	\lim\limits_{x\to -\infty}\frac{x-\cos^2(x)}{3x+\sin(x)}. \end{equation*}

Svolgimento.

La forma indeterminata che si presenta è del tipo \left[\frac{\infty}{\infty}\right]. Mostriamo che in questo caso il teorema 2 non è applicabile, e vediamo quindi qual è il procedimento corretto da applicare per calcolare questo limite. Consideriamo le funzioni f,g\colon \mathbb{R}\to \mathbb{R} definite da

(57)   \begin{equation*} 	f(x)=x-\cos^2(x),\qquad g(x)=3x+\sin(x)\qquad \forall x \in \mathbb{R}. \end{equation*}

Sono derivabili, e le loro derivate hanno legge

(58)   \begin{equation*} 	f'(x)=1-2\sin(x)\cos(x),\qquad g'(x)=3+\cos(x)\qquad \forall x \in \mathbb{R}. \end{equation*}

Osserviamo che g'(x)\neq 0 per ogni x \in \mathbb{R}, però il limite del rapporto delle derivate non esiste. Infatti si ha che la funzione h\colon \mathbb{R}\to \mathbb{R} definita da

(59)   \begin{equation*} 	h(x)\coloneqq\frac{f'(x)}{g'(x)}= \frac{1-2\sin(x)\cos(x)}{3+\cos(x)}\qquad \forall x \in \mathbb{R}, \end{equation*}

è periodica di periodo 2\pi, si annulla per ogni per ogni x=\frac{\pi}{4}+2k\pi con k \in \mathbb{Z}, e vale \frac{1}{4} quando x=2k\pi con k \in \mathbb{Z}. Per cui possiamo concludere che h non ammette limite per x\to -\infty.

Osserviamo che il limite si risolve agilmente con strumenti elementari: infatti, ricordando che per confronto le due funzioni f_1,f_2\colon \mathbb{R}\setminus \{0\}\to \mathbb{R} definite da

(60)   \begin{equation*} 	f_1(x)=x^{-1} \cos^2(x),\qquad f_2(x)=(3x)^{-1}\sin(x)\qquad \forall x \in \mathbb{R}\setminus \{0\}, \end{equation*}

sono infinitesime per x \to -\infty, si ha

(61)   \begin{equation*} 	\lim\limits_{x \to -\infty}\frac{x-\cos^2(x)}{3x+\sin(x)}=\lim\limits_{x \to -\infty}\frac{x\big(1-x^{-1} \cos^2(x)\big)}{3x\big(1+(3x)^{-1}\sin(x)\big)}=\frac{1}{3}. \end{equation*}

 

Esercizio 9   (\bigstar\bigstar\largewhitestar\largewhitestar\largewhitestar). Si calcoli, se esiste, il seguente limite, applicando il teorema di de L’Hôpital:

(62)   \begin{equation*} 	\lim\limits_{x\to 3^+}\left[\log(x^2-4x+3)-\log(x-3)\right]. \end{equation*}

Svolgimento.

Osserviamo che, per come posto, l’esercizio presenta la forma indeterminata \left[\infty-\infty\right] , per cui non potremmo applicare il teorema di de L’Hôpital. Però osserviamo che, sfruttando le proprietà dei logaritmi, si ha

(63)   \begin{equation*} 		\log(x^2-4x+3)-\log(x-3)=\log\left(\frac{x^2-4x+3}{x-3}\right)\qquad \forall x>3, 	\end{equation*}

ed osserviamo che in questo caso l’argomento del logaritmo presenta la forma indeterminata \left[\frac{0}{0}\right] per x\to 3^+. Studiamo quindi il seguente limite

(64)   \begin{equation*} 		\lim\limits_{x\to 3^+}\frac{x^2-4x+3}{x-3}. 	\end{equation*}

Per la continuità della funzione logaritmo, se tale limite esiste ed è pari ad \ell, allora il limite iniziale sarà pari a \log(\ell).

Si considerino le funzioni f,g:\mathbb{R}\to \mathbb{R} definite da

(65)   \begin{equation*} 		f(x)=x^2-4x+3,\qquad g(x)=x-3\qquad \forall x \in \mathbb{R}. 	\end{equation*}

Sono derivabili con derivate definite dalle leggi

(66)   \begin{equation*} 		f'(x)=2x-4,\qquad g'(x)=1\qquad \forall x \in \mathbb{R}. 	\end{equation*}

Il limite del rapporto delle derivate vale

(67)   \begin{equation*} 		\lim\limits_{x\to 3^+}\frac{f'(x)}{g'(x)}=\lim\limits_{x\to 3^+} \frac{2x-4}{1}=2. 	\end{equation*}

Per cui, applicando il teorema 1 si ha

(68)   \begin{equation*} 		\lim\limits_{x\to 3^+}\frac{x^2-4x+3}{x-3}=2. 	\end{equation*}

Quindi possiamo concludere che

(69)   \begin{equation*} 		\lim\limits_{x\to 3^+}\left[\log(x^2-4x+3)-\log(x-3)\right]=\lim\limits_{x\to 3^+}\log\left(\frac{x^2-4x+3}{x-3}\right)=\log(2). 	\end{equation*}

Osserviamo che un altro modo per ottenere lo stesso risultato è scomporre nel seguente modo l’argomento del logaritmo

(70)   \begin{equation*} 		\frac{x^2-4x+3}{x-3}=\frac{(x-1)(x-3)}{x-3}=x-1\qquad \forall x \neq 3, 	\end{equation*}

che ovviamente tende a 2 per x \to 3^+.

 

Esercizio 10   (\bigstar\bigstar\largewhitestar\largewhitestar\largewhitestar). Si calcoli, se esiste, il seguente limite, applicando il teorema di de L’Hôpital:

(71)   \begin{equation*} 	\lim\limits_{x\to \frac{\pi}{4}}\tan(2x)\log(\sin(2x)). \end{equation*}

Svolgimento.

Osserviamo che in questo caso non possiamo applicare direttamente il teorema di de L’Hôpital, in quanto la forma indeterminata che si presenta è del tipo \left0\cdot \infty\right: infatti la tangente in valore assoluto tende a +\infty, mentre l’argomento del logaritmo tende ad 1, e quindi il logaritmo tende a 0. Possiamo manipolare in diversi modi il limite per ricondurci ad una forma indeterminata del tipo \left[\frac{0}{0}\right] o \left[\frac{\infty}{\infty}\right]; un modo di procedere è il seguente

(72)   \begin{equation*} 		\lim\limits_{x\to \frac{\pi}{4}}\tan(2x)\log(\sin(2x))=\lim\limits_{x\to \frac{\pi}{4}}\frac{\sin(2x)\log(\sin(2x))}{\cos(2x)}, 	\end{equation*}

ottenendo così una forma indeterminata del tipo \left[\frac{0}{0}\right]. Osserviamo innanzitutto che, poichè \sin(2x)\to1 quando x\to \frac{\pi}{4}, il limite precedente è uguale al seguente limite

(73)   \begin{equation*} 	\lim\limits_{x\to \frac{\pi}{4}}\frac{\log(\sin(2x))}{\cos(2x)}. 	\end{equation*}

Consideriamo quindi le funzioni f,g\colon(0,\frac{\pi}{2})\to\mathbb{R} definite da

(74)   \begin{equation*} 		f(x)=\log(\sin(2x)),\qquad g(x)=\cos(2x)\qquad \forall x 	\in  \left(0,\frac{\pi}{2}\right). 	\end{equation*}

Queste funzioni sono derivabili, e le loro derivate sono

(75)   \begin{equation*} 		f'(x)=\frac{2\cos(2x)}{\sin(2x)},\qquad g'(x)=-2\sin(2x)\qquad \forall x \in \left(0,\frac{\pi}{2}\right). 	\end{equation*}

In particolare osserviamo che g'(x)\neq 0. Il calcolo del limite del rapporto delle derivate ci dà

(76)   \begin{equation*} 		\lim\limits_{x\to \frac{\pi}{4}}\frac{f'(x)}{g'(x)}=	\lim\limits_{x\to \frac{\pi}{4}}\frac{2\cos(2x)}{-2\sin^2(2x)}=0. 	\end{equation*}

Possiamo quindi applicare il teorema 1, che fornisce

(77)   \begin{equation*} 		\lim\limits_{x\to \frac{\pi}{4}}\tan(2x)\log(\sin(2x))=0. 	\end{equation*}

 

Esercizio 11   (\bigstar\bigstar\largewhitestar\largewhitestar\largewhitestar). Si calcoli, se esiste, il seguente limite, applicando il teorema di de L’Hôpital:

(78)   \begin{equation*} 	\lim\limits_{x\to 0}\frac{(x+4)^{x+1}-4}{x}. \end{equation*}

Svolgimento.

Osserviamo innanzitutto che ci troviamo di fronte ad una forma indeterminata del tipo \left[\frac{0}{0}\right]. Consideriamo quindi le funzioni f,g\colon (-1,+\infty) \to \mathbb{R} definite da

(79)   \begin{equation*} 		f(x)=(x+4)^{x+1}-4 ,\qquad g(x)= x\qquad \forall x \in (-1,+\infty). 	\end{equation*}

Sono entrambe funzioni derivabili. Per ottenere l’espressione di f'(x) applichiamo la regola della catena, osservando che possiamo riscrivere la funzione f nel seguente modo

(80)   \begin{equation*} 		f(x)=\mathrm{e}^{(x+1)\log(x+4)}-4\qquad \forall x \in (-1,+\infty). 	\end{equation*}

Le derivate di f e g sono quindi

(81)   \begin{equation*} 		f'(x)=  \left(\log(x+4)+\frac{x+1}{x+4}\right)\left(x+4\right)^{x+1},\qquad g'(x)=1\qquad \forall x \in (-1,+\infty). 	\end{equation*}

Per applicare il teorema 1 non ci resta che studiare il limite del rapporto delle derivate:

(82)   \begin{equation*} 		\lim\limits_{x\to 0}\frac{f'(x)}{g'(x)}=\lim\limits_{x\to 0}	\left(\log(x+4)+\frac{x+1}{x+4}\right)\left(x+4\right)^{x+1}=4\log(4)+1. 	\end{equation*}

Dal teorema 1 segue che

(83)   \begin{equation*} 		\lim\limits_{x\to 0}\frac{(x+4)^{x+1}-4}{x}=4\log(4)+1. 	\end{equation*}

 

Esercizio 12   (\bigstar\bigstar\largewhitestar\largewhitestar\largewhitestar). Si calcoli, se esiste, il seguente limite, applicando il teorema di de L’Hôpital:

(84)   \begin{equation*} 	\lim\limits_{x\to 0^+}\frac{\cos(\sqrt[3]{x})-\sqrt[3]{\cos(x)}}{x^2}. \end{equation*}

Svolgimento.

Questo esercizio è tratto da [2, Esercizio 7.158].

La forma indeterminata che si presenta è del tipo \left[\frac{0}{0}\right]. Consideriamo quindi le funzioni f,g\colon \left(0,\frac{\pi}{2}\right)\to \mathbb{R} definite da

(85)   \begin{equation*} 		f(x)=\cos(\sqrt[3]{x})-\sqrt[3]{\cos(x)},\qquad g(x)=x^2\qquad \forall x \in \left(0,\frac{\pi}{2}\right), 	\end{equation*}

derivabili con derivate

(86)   \begin{equation*} 		f'(x)=-\sin(\sqrt[3]{x})\frac{1}{3\sqrt[3]{x^2}}+\sin(x)\frac{1}{3\sqrt[3]{\cos^2(x)}},\qquad g'(x)=2x\qquad 	\forall x \in \left(0,\frac{\pi}{2}\right). 	\end{equation*}

Notiamo subito che g'(x)\neq0, e che il limite del rapporto delle derivate vale

(87)   \begin{equation*} 		\lim\limits_{x\to 0^+}	\frac{f'(x)}{g'(x)}=\lim\limits_{x\to 0^+}\left[-\frac{\sin(\sqrt[3]{x})}{2x}\frac{1}{3\sqrt[3]{x^2}}+\frac{\sin(x)}{2x}\frac{1}{3\sqrt[3]{\cos^2(x)}}\right]=-\infty, 	\end{equation*}

in quanto il secondo addendo tende a \frac{1}{6}, mentre il primo addendo diverge: infatti, grazie limite notevole

(88)   \begin{equation*} 		\lim\limits_{t\to 0}\frac{\sin (t)}{t}=1, 	\end{equation*}

si ha

(89)   \begin{equation*} 		\lim_{x\to0^+}-\frac{\sin(\sqrt[3]{x})}{2x}\frac{1}{3\sqrt[3]{x^2}}=\lim_{x\to0^+}-\frac{1}{6}\frac{\sin(\sqrt[3]{x})}{\sqrt[3]{x}}\frac{1}{x^\frac{4}{3}}=-\infty. 	\end{equation*}

Applicando il teorema 1 si ha

(90)   \begin{equation*} 		\lim\limits_{x\to 0^+}\frac{\cos(\sqrt[3]{x})-\sqrt[3]{\cos(x)}}{x^2}=-\infty. 	\end{equation*}

 

Esercizio 13   (\bigstar\bigstar\largewhitestar\largewhitestar\largewhitestar). Si calcoli, se esiste, il seguente limite, applicando il teorema di de L’Hôpital:

(91)   \begin{equation*} 	\lim\limits_{x\to 0}\frac{\sin(\log(3x+1))}{\mathrm{e}^x-3^x}. \end{equation*}

Svolgimento.

Questo esercizio è tratto da [1, Esercizio 8.17].

Studiamo questa forma indeterminata del tipo \left[\frac{0}{0}\right]. Consideriamo quindi le funzioni f,g\colon \left(-\frac{1}{3},\frac{1}{3}\right)\to \mathbb{R} definite da

(92)   \begin{equation*} 	f(x)=\sin(\log(3x+1)),\qquad g(x)=\mathrm{e}^x-3^x\qquad \forall x \in \left(-\frac{1}{3},\frac{1}{3}\right).  \end{equation*}

Sono funzioni derivabili con derivate

(93)   \begin{equation*} 	f'(x)=\cos(\log(3x+1))\frac{3}{3x+1},\qquad g'(x)=e^x-3^x\log(3)\qquad \forall x \in \left(-\frac{1}{3},\frac{1}{3}\right).	  \end{equation*}

Osserviamo che g'(0)\neq 0, per cui per il teorema di permanenza del segno g' non si annulla in un opportuno intorno di 0. Non ci resta quindi che studiare il limite del rapporto delle derivate

(94)   \begin{equation*} 	\lim\limits_{x\to 0}\frac{f'(x)}{g'(x)}=\lim\limits_{x\to 0}	\frac{\cos(\log(3x+1))}{e^x-3^x\log(3)}\frac{3}{3x+1}=\frac{3}{1-\log(3)}. \end{equation*}

Possiamo quindi applicare il teorema teorema 1, che ci dà

(95)   \begin{equation*} 	\lim\limits_{x\to 0} \frac{\sin(\log(3x+1))}{\mathrm{e}^{x}-3^x}=\frac{3}{1-\log(3)}. \end{equation*}

 

Esercizio 14   (\bigstar\bigstar\bigstar\largewhitestar\largewhitestar). Si calcoli, se esiste, il seguente limite, applicando il teorema di de L’Hôpital:

(96)   \begin{equation*} 	\lim_{x \to 4^+} \left ( \frac{x - 5}{x-4} - \frac{1}{\log(x-3)} \right ). \end{equation*}

Svolgimento.

È una forma indeterminata [\infty-\infty]. Scriviamo il limite come

(97)   \begin{equation*} 	\lim_{x \to 4^+} \left ( \frac{x - 5}{x-4} - \frac{1}{\log(x-3)} \right )=\lim_{x \to 4^+}\frac{(x-5)\log(x-3) - (x-4)}{(x-4)\log(x-3)}, \end{equation*}

in modo da avere una forma indeterminata del tipo \left[\frac{0}{0}\right]. Consideriamo quindi le funzioni f,g\colon (4,+\infty)\to \mathbb{R} definite da

(98)   \begin{equation*} 	f(x)=(x-5)\log(x-3)-(x-4),\qquad g(x)=(x-4)\log(x-3)\qquad \forall x \in (4,+\infty). \end{equation*}

Queste sono derivabili con derivate

(99)   \begin{equation*} 	f'(x)=\log(x-3)+\frac{(x-5)}{(x-3)}-1,\qquad g'(x)=\log(x-3)+\frac{(x-4)}{(x-3)}\qquad \forall x \in (4,+\infty). \end{equation*}

Osserviamo che i due addendi che appaiono nella legge di g' sono entrambi positivi per ogni x \in (4,+\infty), e quindi g'(x)\neq0. Per applicare il teorema teorema 1 non ci resta che analizzare il limite del rapporto delle derivate

(100)   \begin{equation*} 	\lim_{x \to 4^+}\frac{f'(x)}{g'(x)}=\lim_{x \to 4^+}\frac{\log(x-3)+\frac{(x-5)}{(x-3)}-1}{\log(x-3)+\frac{(x-4)}{(x-3)}}=-\infty. \end{equation*}

Possiamo quindi concludere che

(101)   \begin{equation*} 	\lim_{x\to 4^+}\left ( \frac{x - 5}{x-4} - \frac{1}{\log(x-3)} \right )=-\infty. \end{equation*}

 

Esercizio 15   (\bigstar\bigstar\bigstar\largewhitestar\largewhitestar). Si calcoli, se esiste, il seguente limite, applicando il teorema di de L’Hôpital:

(102)   \begin{equation*} 	\lim\limits_{x\to 0}(\sin(x))^{2x}. \end{equation*}

Svolgimento.

Si osservi innanzitutto che il limite ha senso solo per x \to 0^+, in quanto la base dell’esponenziale deve essere positiva. Anche in questo caso abbiamo una forma indeterminata non trattata dal teorema di de l’Hôpital, ovvero la forma indeterminata del tipo \left[0^0\right]. Osserviamo che sfruttando esponenziali e logaritmi possiamo scrivere

(103)   \begin{equation*} 	\lim\limits_{x\to 0^+}(\sin(x))^{2x}= \lim\limits_{x\to 0^+} \mathrm{e}^{2x\log(\sin(x))}. \end{equation*}

La forma indeterminata \left[0\cdot \infty\right] all’esponente si riconduce ad una forma indeterminata \left[\frac{\infty}{\infty}\right] nel seguente modo:

(104)   \begin{equation*} 	\lim\limits_{x\to 0^+}2x\log(\sin(x))=\lim\limits_{x\to 0^+}\frac{\log(\sin(x))}{\frac{1}{2x}}. \end{equation*}

Notiamo che è una scrittura lecita in quanto 2x\neq 0 per ogni x \neq 0, e quindi la frazione al denominatore ha senso.

Consideriamo quindi le funzioni f,g\colon(0,\pi)\to \mathbb{R} definite da

(105)   \begin{equation*} 	f(x)=\log(\sin(x)),\qquad g(x)=\frac{1}{2x}\qquad \forall x \in (0,\pi). \end{equation*}

Sono derivabili con derivate pari a

(106)   \begin{equation*} 	f'(x)=\frac{\cos(x)}{\sin(x)}, \qquad g'(x)= -\frac{1}{2x^2}\qquad \forall x \in (0,\pi). \end{equation*}

Possiamo scrivere il limite del rapporto delle derivate come

(107)   \begin{equation*} 	\lim\limits_{x\to 0^+}\frac{f'(x)}{g'(x)}=\lim\limits_{x\to 0^+}\frac{-2x\cos(x)}{\frac{\sin(x)}{x}}=0, \end{equation*}

dove abbiamo sfruttato il limite notevole

(108)   \begin{equation*} 	\lim\limits_{x\to 0^+}\frac{\sin(x)}{x}=1. \end{equation*}

Applicando quindi il teorema teorema 1 si ha

(109)   \begin{equation*} 	\lim\limits_{x\to 0^+}2x\log(\sin(x))=0, \end{equation*}

per cui sfruttando la continuità dell’esponenziale si ha

(110)   \begin{equation*} 	\lim\limits_{x\to 0^+}(\sin(x))^{2x}=\lim\limits_{x\to 0^+} \mathrm{e}^{2x\log(\sin(x))}=1. \end{equation*}

 

Esercizio 16  (\bigstar\bigstar\bigstar\largewhitestar\largewhitestar). Si calcoli, se esiste, il seguente limite, applicando il teorema di de L’Hôpital:

(111)   \begin{equation*} 	\lim\limits_{x\to 0}\frac{2(\tan(x)-\sin(x))}{x^3}. \end{equation*}

Svolgimento.

Siamo in presenza di una forma indeterminata del tipo \left[\frac{0}{0}\right]. Anticipiamo che in questo caso l’uso del teorema di de L’Hôpital non è conveniente, in quanto porta a calcoli più complessi del necessario. Possiamo infatti scrivere il limite precedente come

(112)   \begin{equation*} 	\lim\limits_{x\to 0}\frac{2(\tan(x)-\sin(x))}{x^3}=		\lim\limits_{x\to 0}\frac{\sin(x)}{x}\frac{2(1-\cos(x))}{\cos(x)x^2},	 \end{equation*}

e con limiti notevoli di seno e coseno otteniamo che tale limite è pari ad 1 , visto che \cos(x)\to1 quando x\to 0. Volendo applicare il teorema di de L’Hôpital si ottiene il seguente svolgimento. Consideriamo le funzioni f,g\colon \left(-\frac{\pi}{2},\frac{\pi}{2}\right)\to \mathbb{R} definite da

(113)   \begin{equation*} 	f(x)=2(\tan(x)-\sin(x)),\qquad g(x)=x^3\qquad \forall x \in  \left(-\frac{\pi}{2},\frac{\pi}{2}\right), \end{equation*}

derivabili, con derivate definite da

(114)   \begin{equation*} 	f'(x)=2\left(\frac{1}{\cos^2(x)}-\cos(x)\right),\qquad g'(x)=3x^2\qquad  \forall x \in  \left(-\frac{\pi}{2},\frac{\pi}{2}\right). \end{equation*}

Osserviamo che g' si annulla solo per x=0 per cui siamo nelle ipotesi del teorema teorema 1 se il limite del rapporto delle derivate esiste. Tale limite è

(115)   \begin{equation*}\begin{aligned} 		\lim\limits_{x\to 0}\frac{f'(x)}{g'(x)}=&\lim\limits_{x\to 0} \frac{2}{3x^2}\left(\frac{1}{\cos^2(x)}-\cos(x)\right)=\\=&    \lim\limits_{x\to 0} \frac{2}{3x^2}\left({\frac{1-\cos^3(x)}{\cos^2(x)}}\right)=\\ =& \lim\limits_{x\to 0} \frac{2}{3}\left({\frac{(1-\cos(x))(1+\cos^2(x)+\cos(x))}{x^2\cos^2(x)}}\right), \end{aligned}\end{equation*}

dove abbiamo sfruttato la scomposizione di cubi (a^3-b^3)=(a-b)(a^2+b^2+ab).

Utilizzando il limite notevole

(116)   \begin{equation*} 	\lim\limits_{x\to 0}\frac{1-\cos(x)}{x^2}=\frac{1}{2}, \end{equation*}

otteniamo

(117)   \begin{equation*} 	\lim\limits_{x\to 0} \frac{2}{3}\left({\frac{(1-\cos(x))(1+\cos^2(x)+\cos(x))}{x^2\cos^2(x)}}\right)=1. \end{equation*}

Possiamo quindi applicare il teorema teorema 1, che ci dà

(118)   \begin{equation*} 	\lim\limits_{x\to 0}\frac{2(\tan(x)-\sin(x))}{x^3}=1. \end{equation*}

 

Esercizio 17   (\bigstar\bigstar\bigstar\largewhitestar\largewhitestar). Si calcoli, se esiste, il seguente limite, applicando il teorema di de L’Hôpital:

(119)   \begin{equation*} 	\lim\limits_{x\to 0^+} \frac{(\sin x)^x-1}{x^x-1} \end{equation*}

Svolgimento.

Per studiare questo limite, ci conviene innanzitutto riscriverlo nel seguente modo

(120)   \begin{equation*} 	\lim\limits_{x\to 0^+}\frac{\mathrm{e}^{x\log(\sin (x))}-1}{\mathrm{e}^{x\log(x)}-1}. \end{equation*}

Mostriamo innanzitutto che l’esponente di e al numeratore tende a 0 per x \to 0^+. Applicando il teorema teorema 2 si vede che

(121)   \begin{equation*} 	\lim\limits_{x\to 0^+}x\log(\sin(x))=\lim\limits_{x\to 0^+}\frac{\log(\sin(x))}{\frac{1}{x}}=0. \end{equation*}

Infatti, se consideriamo le funzioni f,g\colon \left(0,\frac{\pi}{2}\right)\to \mathbb{R} definite da

(122)   \begin{equation*} 	f(x)=\log(\sin(x)),\qquad g(x)=\frac{1}{x}\qquad x \in \left(0,\frac{\pi}{2}\right), \end{equation*}

notiamo che sono degli infiniti per x\to 0^+, ed inoltre le loro derivate sono

(123)   \begin{equation*} 	f'(x)=\frac{\cos(x)}{\sin(x)},\qquad g'(x)=\frac{-1}{x^2}\qquad x \in \left(0,\frac{\pi}{2}\right). \end{equation*}

Il limite del rapporto delle derivate è

(124)   \begin{equation*} 	\lim\limits_{x\to0^+}\frac{f'(x)}{g'(x)}=\lim\limits_{x\to 0^+}\frac{-x^2\cos(x)}{\sin(x)}=0, \end{equation*}

dove abbiamo sfruttato il limite notevole

(125)   \begin{equation*} 	\lim\limits_{t \to 0}\frac{\sin(t)}{t}=1. \end{equation*}

Osservato che anche x\log(x)\to 0 per x\to 0^+, riscriviamo il limite (120) nel seguente modo

(126)   \begin{equation*} 	\lim\limits_{x\to 0^+}\frac{\mathrm{e}^{x\log(\sin (x))}-1}{x\log(\sin (x))}\frac{{x\log(x)}}{\mathrm{e}^{x\log(x)}-1}\frac{x\log(\sin (x))}{x\log(x)}. \end{equation*}

Da tale espressione, per il teorema del limite della funzione composta ed il limite notevole dell’esponenziale

(127)   \begin{equation*} 	\lim_{t \to 0^+}\frac{e^t-1}{t}=1, \end{equation*}

otteniamo che il limite di partenza è uguale al seguente limite

(128)   \begin{equation*} 	\lim\limits_{x\to 0^+}\frac{\log(\sin(x))}{\log(x)}. \end{equation*}

Applichiamo nuovamente il teorema teorema 2 alle funzioni f,h\colon \left(0,\frac{\pi}{2}\right)\to \mathbb{R} definite da

(129)   \begin{equation*} 	f(x)=\log(\sin(x)),\qquad h(x)=\log(x)\qquad x \in \left(0,\frac{\pi}{2}\right), \end{equation*}

le cui derivate sono definite da

(130)   \begin{equation*} 	f'(x)=\frac{\cos(x)}{\sin(x)},\qquad h'(x)=\frac{1}{x}\qquad x \in \left(0,\frac{\pi}{2}\right). \end{equation*}

Sfruttando nuovamente il limite notevole (125), si ha che il limite del rapporto delle derivate è

(131)   \begin{equation*} 	\lim\limits_{x\to0^+}\frac{f'(x)}{h'(x)}=\lim\limits_{x\to 0^+}\frac{x\cos(x)}{\sin(x)}=1, \end{equation*}

per cui per il teorema teorema 2 possiamo concludere che

(132)   \begin{equation*} 	\lim\limits_{x\to 0^+}\frac{\log(\sin(x))}{\log(x)}=	\lim\limits_{x\to 0^+}\frac{x\cos(x)}{\sin(x)}=1, \end{equation*}

che per quanto detto in precedenza è pari al valore del limite di partenza.

 

Esercizio 18   (\bigstar\bigstar\bigstar\bigstar\largewhitestar). Si calcoli, se esiste, il seguente limite, applicando il teorema di de L’Hôpital:

(133)   \begin{equation*} 	\lim\limits_{x\to 0^+}\frac{x^2\mathrm{\arctan}(3(x-x^2))}{\sinh(x)-x\cosh(x)}. \end{equation*}

Svolgimento.

Ricordiamo che le funzioni \sinh,\cosh \colon \mathbb{R} \to \mathbb{R} sono definite da

(134)   \begin{equation*} 	\sinh (x) = \frac{e^x-e^{-x}}{2}, 	\quad 	\cosh (x) = \frac{e^x+e^{-x}}{2}		\qquad 	\forall x \in \mathbb{R}. \end{equation*}

In quanto somma e differenza di esponenziali, sia \sinh che \cosh sono derivabili infinite volte e le loro derivate soddisfano

(135)   \begin{equation*} 	\frac{\mathrm{d}}{\mathrm{d}x}\sinh (x)=\frac{e^x+e^{-x}}{2} =	\cosh (x),	\qquad 		\frac{\mathrm{d}}{\mathrm{d}x}\cosh (x) 		= 		\frac{e^x-e^{-x}}{2} 		= 		\sinh (x) 		\qquad 		\forall x \in \mathbb{R}. \end{equation*}

Moltiplicando e dividendo per x l’argomento del limite si ottiene

(136)   \begin{equation*} 	\lim_{x \to 0} \frac{x^2\mathrm{\arctan}(3(x-x^2))}{\sinh(x)-x\cosh(x)} 	= 	\lim_{x \to 0} \frac{x^3}{\sinh (x) - x \cosh (x)} \frac{\arctan(3(x-x^2))}{x}. \end{equation*}

Vogliamo applicare il teorema sul limite del prodotto per i due fattori in (136), di cui calcoliamo separatamente i limiti.

  • Per il secondo fattore in (136) si ha

    (137)   \begin{equation*} 		\lim_{x \to 0} \frac{\arctan(3(x-x^2))}{x} 		= 		\lim_{x \to 0} \frac{\arctan(3(x-x^2))}{3(x-x^2)} \frac{3(x-x^2)}{x} 		= 		3, 	\end{equation*}

    dove si è applicato il limite notevole

    (138)   \begin{equation*} 			\lim_{t \to 0} \frac{\arctan (t)}{t}=1, 		\end{equation*}

    che si può ottenere usando il cambio di variabili t=\tan y e il limite notevole

    (139)   \begin{equation*} 			\lim_{y \to 0} \frac{\tan (y)}{y}=1. 		\end{equation*}

  • Per calcolare il limite del primo fattore in (136), che presenta una forma indeterminata \left \frac{0}{0} \right, usiamo il teorema teorema 1. Consideriamo le funzioni f,g \colon \mathbb{R} \to \mathbb{R} definite da

    (140)   \begin{equation*} 			f(x)=x^3, 			\quad 			g(x)= \sinh (x) - x \cosh (x) 			\qquad 			\forall x \in \mathbb{R}. 		\end{equation*}

    f e g sono derivabili infinite volte in \mathbb{R}. Applicando (135) si ha in particolare

    (141)   \begin{equation*} 			f'(x)=3x^2, 			\quad 			g'(x)= \cosh (x) - \cosh (x) - x \sinh (x) 			= 			-x \sinh (x) 			\qquad 			\forall x \in \mathbb{R}. 		\end{equation*}

    Si osservi che x \sinh (x)=0 se e solo se x=0. Si ha inoltre

    (142)   \begin{equation*} 			\lim_{x \to 0} \frac{f'(x)}{g'(x)} 			= 			\lim_{x \to 0} \frac{-3x}{\sinh (x)} 			= 			\lim_{x \to 0} \frac{-6x}{(e^x - 1) - (e^{-x}-1)} 			= 			\lim_{x \to 0} \frac{-6}{\frac{e^x-1}{x} + \frac{e^{-x}-1}{-x}} 			= 			-3, 		\end{equation*}

    dove nella seconda uguaglianza si è sommato e sottratto 1 al denominatore, mentre nell’ultima si è usato il limite notevole

    (143)   \begin{equation*} 			\lim_{x \to 0}\frac{e^x-1}{x}=1. 		\end{equation*}

    Per il teorema teorema 1 si ha che

    (144)   \begin{equation*} 			\lim_{x \to 0} \frac{f(x)}{g(x)} 			= 			\lim_{x \to 0}  			\frac{x^3}{\sinh (x) - x \cosh (x)} 			= 			-3. 		\end{equation*}

Inserendo (144) e (137) in (136) e applicando il teorema sul limite di un prodotto, si ottiene

(145)   \begin{equation*} 		\lim_{x \to 0} \frac{x^2\mathrm{\arctan}(3(x-x^2))}{\sinh(x)-x\cosh(x)} 		= 		\lim_{x \to 0} \frac{x^3}{\sinh (x) - x \cosh (x)} \frac{\arctan(3(x-x^2))}{x} 		= 		(-3) \cdot (3) 		= 		-9. 	\end{equation*}

 

Esercizio 19   (\bigstar\bigstar\bigstar\bigstar\largewhitestar). Si calcoli, se esiste, il seguente limite, applicando il teorema di de L’Hôpital:

(146)   \begin{equation*} 	\lim_{x \to 1} \frac{(\sqrt[3]{x}-1) \left ( 1- \sin \left (\frac{\pi}{2} x \right )\right )}{\sin^2(\log (x))}. \end{equation*}

Svolgimento.

Il limite presenta una forma indeterminata \left [ \frac{0}{0} \right ]. Osserviamo che la frazione è definita nell’insieme (1-a,1+a) \setminus \{1\}, dove a \in (0,1) è un numero sufficientemente piccolo. Infatti, poiché la funione \sin è strettamente crescente in un intorno di 0 e la funzione \log è strettamente crescente e continua, esiste a \in (0,1) tale che

(147)   \begin{equation*} 		\log (x)\in (-\pi,\pi) \setminus \{0\} 		\qquad 		\forall x \in (1-a,1+a) \setminus \{1\}. 	\end{equation*}

Quindi \sin^2(\log (x)) non si annulla in (1-a,1+a) \setminus \{1\} e la frazione è ben definita.

Moltiplicando e dividendo la frazione per \log^2 (x), si ottiene

(148)   \begin{equation*} 		\frac{(\sqrt[3]{x}-1) \left ( 1- \sin \left (\frac{\pi}{2} x \right )\right )}{\log^2 (x)} \frac{\log^2 (x)}{\sin^2(\log (x))} 		\qquad 		\forall x \in (1-a,1+a) \setminus \{1\}. 	\end{equation*}

Calcoliamo separatamente i limiti dei due fattori, per poi applicare il teorema sul limite di un prodotto.

  • Per il secondo fattore, utilizzando la sostituzione t=\log (x) e il limite notevole

    (149)   \begin{equation*} 			\lim\limits_{t \to 0} \frac{t}{\sin (t)}=1, 		\end{equation*}

    si ottiene

    (150)   \begin{equation*} 			\lim_{x \to 1} \frac{\log^2 (x)}{\sin^2(\log (x))} 			= 			\lim_{t \to 0} \frac{t^2}{\sin^2 (t)} 			= 			1. 		\end{equation*}

     

  • Per il primo fattore, applichiamo il teorema teorema 1. Consideriamo le funzioni f,g \colon (1-a,1+a) \to \mathbb{R} definite da  

    (151)   \begin{equation*} 			f(x)=(\sqrt[3]{x}-1) \left ( 1- \sin \left (\frac{\pi}{2} x \right )\right ). 			\quad 			g(x)=\log^2 (x) 			\qquad 			\forall x \in (1-a,1+a). 		\end{equation*}

    Esse sono derivabili con derivate pari a

    (152)   \begin{equation*} 			f'(x)= \frac{1- \sin \left (\frac{\pi}{2} x \right )}{3 \sqrt[3]{x^2}} - (\sqrt[3]{x}-1) \frac{\pi}{2} \cos \left (\frac{\pi}{2} x \right ), 			\quad 			g'(x)=\frac{2 \log (x)}{x} 			\qquad 			\forall x \in (1-a,1+a). 		\end{equation*}

    Si vede che g' si annulla solo per x=1. Si ha però

    (153)   \begin{equation*} 			\lim_{x \to 1} \frac{f'(x)}{g'(x)} 			= 			\lim_{x \to 1} 			\frac{\dfrac{\sqrt[3]{x} \left (1- \sin \left (\frac{\pi}{2} x \right )\right) }{3} - x(\sqrt[3]{x}-1) \frac{\pi}{2} \cos \left (\frac{\pi}{2} x \right )}{2 \log (x)}, 		\end{equation*}

    che è di nuovo una forma indeterminata \left[\frac{0}{0}\right] e per risolverla, applichiamo di nuovo il teorema teorema 1. Le funzioni \phi,\psi \colon (1-a,1+a) \to \mathbb{R} definite rispettivamente dal numeratore e dal denominatore della frazione in (153) sono derivabili in (1-a,1+a) e le loro derivate sono pari a

    (154)   \begin{equation*} 			\begin{gathered} 				\phi'(x) 				= 				\frac{1- \sin \left (\frac{\pi}{2} x \right )}{9 \sqrt[3]{x^2}} 				- \left ( \frac{\sqrt[3]{x} }{3} + \frac{\sqrt[3]{x}-1}{2} \right ) 				\pi \cos \left (\frac{\pi}{2} x \right ) 				+ x(\sqrt[3]{x}-1) \frac{\pi^2}{4} \sin \left (\frac{\pi}{2} x \right ) 				\qquad 				\forall x \in (1-a,1+a), 				\\[5pt] 				\psi'(x) 				= 				\frac{2}{x} 				\qquad 				\forall x \in (1-a,1+a). 			\end{gathered} 		\end{equation*}

    Poiché \lim\limits_{x \to 1} \phi'(x)=0 e \lim\limits_{x \to 1} \psi'(x)=2, si ha

    (155)   \begin{equation*} 			\lim_{x \to 1} 			\frac{\phi'(x)}{\psi'(x)} 			= 			0. 		\end{equation*}

    Da ciò, applicando il teorema teorema 1 alle funzioni \phi,\psi, segue quindi che il limite in (153) è pari a

    (156)   \begin{equation*} 			\lim_{x \to 1}  \frac{f'(x)}{g'(x)} 			= 			0. 		\end{equation*}

    Da tale risultato, applicando ancora il teorema teorema 1 alle funzioni f,g, si ottiene

    (157)   \begin{equation*} 			\lim_{x \to 1} \frac{f(x)}{g(x)} 			= 			\lim_{x \to 1} \frac{(\sqrt[3]{x}-1) \left ( 1- \sin \left (\frac{\pi}{2} x \right )\right )}{\log^2 (x)} 			= 			0. 		\end{equation*}

Tenendo conto di (157) e (150) e applicando il teorema sul limite del prodotto, si ha

(158)   \begin{equation*} 		\lim_{x \to 1} \frac{(\sqrt[3]{x}-1) \left ( 1- \sin \left (\frac{\pi}{2} x \right )\right )}{\sin^2(\log (x))} 		= 		\lim_{x \to 1}  		\frac{(\sqrt[3]{x}-1) \left ( 1- \sin \left (\frac{\pi}{2} x \right )\right )}{\log^2 (x)} \frac{\log^2 (x)}{\sin^2(\log (x))} 		= 		0. 	\end{equation*}

 

Esercizio 20   (\bigstar\bigstar\bigstar\bigstar\largewhitestar). Si calcoli, se esiste, il seguente limite, applicando il teorema di de L’Hôpital:

(159)   \begin{equation*} 	\lim_{x \to 1} \frac{ (x-1)^2\left (\arctan (x) - \frac{\pi}{4}\right )}{\sin \left (\pi x \right ) \tan^2(\pi x)}. \end{equation*}

Svolgimento.

Si tratta di una forma indeterminata \left [ \frac{0}{0} \right ]. Scriviamo la frazione come prodotto di due frazioni

(160)   \begin{equation*} 		\lim_{x \to 1} \frac{ (x-1)^2\left (\arctan (x) - \frac{\pi}{4}\right )}{\sin \left (\pi x \right ) \tan^2(\pi x)} 		= 		\lim_{x \to 1} \left ( \frac{(x-1)^2}{\tan^2 (\pi x)} \frac{\arctan (x) - \frac{\pi}{4}}{\sin (\pi x)} \right ). 	\end{equation*}

Vogliamo calcolare separatamente il limite dei due fattori.

  • Per il primo fattore in (160), mediante la sostituzione t=x-1

    (161)   \begin{equation*} 			\lim_{x \to 1} \frac{(x-1)^2}{\tan^2 (\pi x)} 			= 			\lim_{t \to 0} \frac{t^2}{\tan^2 (\pi t + \pi)} 			= 			\lim_{t \to 0} \frac{t^2}{\tan^2 (\pi t)} 			= 			\lim_{t \to 0} \frac{t^2 \cos^2 (\pi t)}{\sin^2(\pi t)} 			= 			\frac{1}{\pi^2} , 		\end{equation*}

    dove nella seconda uguaglianza si è usata la periodicità di periodo \pi della tangente, mentre nell’ultima si è usato il limite notevole

    (162)   \begin{equation*} 			\lim_{y \to 0} \frac{y}{\sin (y)} = 1. 		\end{equation*}

  • Per calcolare il limite del secondo fattore in (160), che a sua volta è una forma indeterminata \left [ \frac{0}{0} \right ], applichiamo il teorema teorema 1. Si considerino le funzioni f,g \colon \left ( \frac{1}{2},\frac{3}{2} \right ) \to \mathbb{R} definite da

    (163)   \begin{equation*} 			f(x)=\arctan (x) - \frac{\pi}{4}, 			\quad 			g(x)= \sin \left (\pi x \right ) 			\qquad 			\forall x \in \mathbb{R}. 		\end{equation*}

    Esse sono derivabili con derivate pari a

    (164)   \begin{equation*} 			f'(x)= \frac{1}{1+x^2}, 			\quad 			g'(x) = \pi \cos(\pi x) 			\qquad 			\forall x \in \left ( \frac{1}{2},\frac{3}{2} \right ). 		\end{equation*}

    Osserviamo che g'(x) \neq 0 per ogni x \in \left ( \frac{1}{2},\frac{3}{2} \right ) \setminus \{1\} e che si ha

    (165)   \begin{equation*} 			\lim_{x \to 1} \frac{f'(x)}{g'(x)} 			= 			\lim_{x \to 1} \frac{1}{(1+x^2)\pi \cos (\pi x)} 			= 			- \frac{1}{2\pi}. 		\end{equation*}

    Da ciò e dal teorema teorema 1 segue che

    (166)   \begin{equation*} 			\lim_{x \to 1} \frac{f(x)}{g(x)} 			= 			\lim_{x \to 1} 			\frac{\arctan (x) - \frac{\pi}{4}}{\sin (\pi x)}  			= 			- \frac{1}{2\pi}. 		\end{equation*}

Inserendo (161) e (166) in (160) e applicando il teorema sul limite del prodotto, si ottiene

(167)   \begin{equation*} 		\lim_{x \to 1} \frac{ (x-1)^2\left(\arctan (x) - \frac{\pi}{4}\right)}{\sin \left (\pi x \right ) \tan^2(\pi x)} 		= 		\lim_{x \to 1} \left ( \frac{(x-1)^2}{\tan^2 (\pi x)} \frac{\arctan (x) - \frac{\pi}{4}}{\sin (\pi x)} \right ) 		= 		-\frac{1}{2\pi^3}. 	\end{equation*}

Si noti che si sarebbe teoricamente potuto risolvere il limite anche applicando il teorema teorema 1 alla frazione iniziale, ma ciò avrebbe portato a calcoli sensibilmente più complessi. Inoltre, ci si può rendere conto che non sarebbe bastata una sola applicazione del teorema teorema 1, in quanto il limite del rapporto delle derivate fornisce nuovamente una forma indeterminata \left [ \frac{0}{0} \right ]. Si sarebbe dunque dovuto calcolare le derivate seconde e terze di numeratore e denominatore, complicando ulteriormente i calcoli. Nello svolgimento proposto, invece, si è separata la frazione nel prodotto di due fattori e, studiando il primo con altri strumenti, il procedimento risolutivo è stato molto semplificato. Questo esempio mostra come sia spesso conveniente integrare strumenti diversi nella risoluzione di un esercizio, in quanto l’applicazione meccanica di un singolo teorema può portare a soluzioni lunghe e complicate.

 

Esercizio 21   (\bigstar\bigstar\bigstar\bigstar\bigstar). Si calcoli, se esiste, il seguente limite, applicando il teorema di de L’Hôpital:

(168)   \begin{equation*} 	\lim_{x \to +\infty} \frac{x^x\log (x)}{\left (x+\frac{1}{x}\right )^{x-\frac{1}{x}} \log(1 + x \arctan (x))}. \end{equation*}

Svolgimento.

Il limite è una forma indeterminata \left [ \frac{\infty}{\infty} \right ]. La struttura della frazione suggerisce di scriverla come prodotto di due frazioni:

(169)   \begin{equation*} 		\lim_{x \to +\infty} \frac{x^x\log (x)}{\left (x+\frac{1}{x}\right )^{x-\frac{1}{x}} \log(1 + x \arctan (x))} 		= 		\lim_{x \to + \infty} \left ( \frac{x^x}{\left (x+\frac{1}{x}\right )^{x-\frac{1}{x}}} \,\frac{\log (x)}{\log(1 + x \arctan (x))} \right ). 	\end{equation*}

Calcoliamo separatamente il limite dei due fattori.  

  • Per il primo fattore in (169), si ha

    (170)   \begin{equation*} 			\lim_{x \to + \infty} \frac{x^x}{\left (x+\frac{1}{x}\right )^{x-\frac{1}{x}}}  			= 			\lim_{x \to + \infty}  \frac{e^{x \log (x)}}{e^{(x-\frac{1}{x}) \log (x+\frac{1}{x})}} 			= 			\lim_{x \to + \infty} e^{x \log (x) - (x-\frac{1}{x}) \log (x+\frac{1}{x})}. 		\end{equation*}

    All’esponente abbiamo una forma indeterminata [\infty-\infty]. Per x >1 vale

    (171)   \begin{equation*} 			\begin{split} 				x \log (x) - \left (x-\frac{1}{x}\right ) \log \left (x+\frac{1}{x}\right ) 				= & 				x \left (\log (x) - \log \left (x+\frac{1}{x}\right )\right ) 				+ \frac{1}{x} \log \left (x+\frac{1}{x}\right ) 				\\ 				= & 				\frac{\log \left ( \frac{x}{x+ \frac{1}{x}}\right )}{\frac{1}{x}} 				+ 				\frac{\log \left (x+\frac{1}{x}\right )}{x}. 			\end{split} 		\end{equation*}

    Studiamo quindi separatamente i limiti dei due addendi nell’ultimo membro di (171).

      1. Per il primo termine in (171) si ha

    (172)   \begin{equation*} 				\lim_{x \to + \infty} \frac{\log \left ( \frac{x}{x+ \frac{1}{x}}\right )}{\frac{1}{x}} 				= 				\lim_{x \to + \infty}  \frac{\log \left ( 1 - \frac{1}{x\left( x + \frac{1}{x} \right )} \right )}{\frac{1}{x\left( x + \frac{1}{x} \right )}} \frac{\frac{1}{x\left( x + \frac{1}{x} \right )}}{\frac{1}{x}} 				= 				\lim_{x \to + \infty}  \frac{\log \left ( 1 - \frac{1}{x\left( x + \frac{1}{x} \right )} \right )}{\frac{1}{x\left( x + \frac{1}{x} \right )}} 				\frac{1}{ x + \frac{1}{x} } 				= 				0, 			\end{equation*}

    dove nella prima uguaglianza abbiamo moltiplicato e diviso per \frac{1}{x\left( x + \frac{1}{x} \right )}, mentre nell’ultima abbiamo usato il fatto che

    (173)   \begin{equation*} 				\lim_{x \to +\infty} \frac{1}{x\left( x + \frac{1}{x} \right )}=0,  			\end{equation*}

    ed applicato il limite notevole

    (174)   \begin{equation*} 				\lim_{t \to 0} \frac{\log(1+t)}{t}=1. 			\end{equation*}

    2. Per il secondo termine in (171), applichiamo il teorema 1 alle funzioni f,g \colon (0,+\infty) \to \mathbb{R} definite da

    (175)   \begin{equation*} 				f(x)= \log \left (x+\frac{1}{x}\right ), 				\quad 				g(x) = x 				\qquad 				\forall x \in (0,+\infty). 			\end{equation*}

    Esse sono derivabili con derivate pari a

    (176)   \begin{equation*} 				f'(x)= \frac{1 - \frac{1}{x^2}}{x+ \frac{1}{x}}, 				\quad 				g'(x) = 1 				\qquad 				\forall x \in (0,+\infty). 			\end{equation*}

    Ovviamente g' non si annulla mai e vale

    (177)   \begin{equation*} 				\lim_{x \to + \infty} \frac{f'(x)}{g'(x)} 				= 				\lim_{x \to + \infty} \frac{1 - \frac{1}{x^2}}{x+ \frac{1}{x}} 				= 				0. 			\end{equation*}

    Per il teorema 1 si ha dunque

    (178)   \begin{equation*} 				\lim_{x \to + \infty} \frac{f(x)}{g(x)} 				= 				\lim_{x \to + \infty} \frac{\log \left (x+\frac{1}{x}\right )}{x} 				= 				0. \end{equation*}

    Inserendo (172) e (178) in (171), dal teorema sul limite della somma si ottiene

    (179)   \begin{equation*} 			\lim_{x \to + \infty} \left ( x \log (x) - \left (x-\frac{1}{x}\right ) \log \left (x+\frac{1}{x}\right ) \right ) 			= 			0. 		\end{equation*}

    Per la continuità dell’esponenziale in 0 e per il teorema sul limite delle funzioni composte e usando (170), si ha dunque

    (180)   \begin{equation*} 			\lim_{x \to + \infty} \frac{x^x}{\left (x+\frac{1}{x}\right )^{x-\frac{1}{x}}}  			= 			\lim_{x \to + \infty} e^{x \log (x) - \left (x-\frac{1}{x}\right ) \log \left (x+\frac{1}{x}\right ) } 			= 			1. 		\end{equation*}

  •  

  • Per il secondo fattore in (169), si utilizza il teorema 1. Si considerino le funzioni \phi,\psi \colon (0,+\infty) \to \mathbb{R} definite da  

    (181)   \begin{equation*} 			\phi(x) = \log (x), 			\quad 			\psi(x) = \log(1+ x \arctan (x)) 			\qquad 			\forall x \in (0,+\infty). 		\end{equation*}

    Esse sono derivabili con derivate pari a

    (182)   \begin{equation*} 			\phi'(x) = \frac{1}{x}, 			\quad 			\psi'(x) = \frac{\arctan \left(x + \frac{x}{1+x^2}	\right)}{1+ x \arctan (x)} 			\qquad 			\forall x \in (0,+\infty). 		\end{equation*}

    Osserviamo che \psi'(x)>0 per ogni x \in (0,+\infty), ed inoltre si ha

    (183)   \begin{equation*} 			\lim_{x \to + \infty} \frac{\phi'(x)}{\psi'(x)} 			= 			\lim_{x \to + \infty} \frac{1+ x \arctan (x)}{x  \arctan \left(x + \frac{x}{1+x^2}\right )} 			= 			\lim_{x \to + \infty} \frac{\arctan\left( x + \frac{1}{x}\right)}{\arctan \left(x +  \frac{x}{1+x^2}\right)} 			= 			1. 		\end{equation*}

    Da ciò, per il teorema 1, vale

    (184)   \begin{equation*} 			 \lim_{x \to + \infty} \frac{\phi(x)}{\psi(x)} 			= 			\lim_{x \to +\infty} \frac{\log (x)}{\log(1 + x \arctan (x))} 			= 			1. 		\end{equation*}

Da (169), (180), (184) e dal teorema sul limite del prodotto, segue che

(185)   \begin{equation*} 		\lim_{x \to +\infty} \frac{x^x\log (x)}{\left (x+\frac{1}{x}\right )^{x-\frac{1}{x}} \log(1 + x \arctan (x))} 		= 		\lim_{x \to + \infty} \left ( \frac{x^x}{\left (x+\frac{1}{x}\right )^{x-\frac{1}{x}}} \,\frac{\log (x)}{\log(1 + x \arctan (x))} \right ) 		= 		1. 	\end{equation*}

 

Riferimenti bibliografici

[1] G. De Marco, C. Mariconda, Esercizi di calcolo in una variabile., Zanichelli, 2001.

[2] E. Giusti, Esercizi e complementi di analisi matematica (Vol. 1), Bollati Boringhieri, 1991.

[3] Qui Si Risolve, I teoremi di de L’Hôpital .

 
 

Tutta la teoria di analisi matematica

Leggi...

  1. Teoria Insiemi
  2. Il metodo della diagonale di Cantor
  3. Logica elementare
  4. Densità dei numeri razionali nei numeri reali
  5. Insiemi Numerici \left(\mathbb{N},\, \mathbb{Z},\, \mathbb{Q}\right)
  6. Il principio di induzione
  7. Gli assiomi di Peano
  8. L’insieme dei numeri reali: costruzione e applicazioni
  9. Concetti Fondamentali della Retta Reale: Sintesi Teorica
  10. Costruzioni alternative di \mathbb{R}
  11. Binomio di Newton
  12. Spazi metrici, un’introduzione
  13. Disuguaglianza di Bernoulli
  14. Disuguaglianza triangolare
  15. Teoria sulle funzioni
  16. Funzioni elementari: algebriche, esponenziali e logaritmiche
  17. Funzioni elementari: trigonometriche e iperboliche
  18. Funzioni goniometriche: la guida essenziale
  19. Teorema di Bolzano-Weierstrass per le successioni
  20. Criterio del rapporto per le successioni
  21. Definizione e proprietà del numero di Nepero
  22. Limite di una successione monotona
  23. Successioni di Cauchy
  24. Il teorema ponte
  25. Teoria sui limiti
  26. Simboli di Landau
  27. Funzioni continue – Teoria
  28. Il teorema di Weierstrass
  29. Il teorema dei valori intermedi
  30. Il teorema della permanenza del segno
  31. Il teorema di Heine-Cantor
  32. Il teorema di esistenza degli zeri
  33. Il metodo di bisezione
  34. Teorema ponte versione per le funzioni continue
  35. Discontinuità di funzioni monotone
  36. Continuità della funzione inversa
  37. Teorema delle contrazioni o Teorema di punto fisso di Banach-Caccioppoli
  38. Teoria sulle derivate
  39. Calcolo delle derivate: la guida pratica
  40. Teoria sulle funzioni convesse
  41. Il teorema di Darboux
  42. I teoremi di de l’Hôpital
  43. Teorema di Fermat
  44. Teoremi di Rolle e Lagrange
  45. Il teorema di Cauchy
  46. Espansione di Taylor: teoria, esempi e applicazioni pratiche
  47. Polinomi di Taylor nei limiti: istruzioni per l’uso
  48. Integrali definiti e indefiniti
  49. Teorema fondamentale del calcolo integrale (approfondimento)
  50. Integrali ricorsivi
  51. Formule del trapezio, rettangolo e Cavalieri-Simpson
  52. Teoria sugli integrali impropri
  53. Funzioni integrali – Teoria
  54. Introduzione ai numeri complessi – Volume 1 (per un corso di ingegneria — versione semplificata)
  55. Introduzione ai numeri complessi – Volume 1 (per un corso di matematica o fisica)
  56. Serie numeriche: la guida completa
  57. Successioni di funzioni – Teoria
  58. Teoremi sulle successioni di funzioni
    1. 58a. Criterio di Cauchy per la convergenza uniforme
    2. 58b. Limite uniforme di funzioni continue
    3. 58c. Passaggio al limite sotto il segno di integrale
    4. 58d. Limite uniforme di funzioni derivabili
    5. 58e. Piccolo teorema del Dini
    6. 58f. Procedura diagonale e teorema di Ascoli-Arzela
  59. Serie di funzioni – Teoria
  60. Serie di potenze – Teoria
  61. Serie di Fourier – Teoria e applicazioni
  62. Integrali multipli — Parte 1 (teoria)
  63. Integrali multipli — Parte 2 (teoria e esercizi misti)
  64. Regola della Catena — Teoria ed esempi.
  65. Jacobiano associato al cambiamento di coordinate sferiche
  66. Guida ai Massimi e Minimi: Tecniche e Teoria nelle Funzioni Multivariabili
  67. Operatore di Laplace o Laplaciano
  68. Teoria equazioni differenziali
  69. Equazione di Eulero
  70. Teoria ed esercizi sulla funzione Gamma di Eulero
  71. Teoria ed esercizi sulla funzione Beta
  72. Approfondimento numeri complessi
  73. Diverse formulazioni dell’assioma di completezza
  74. Numeri di Delannoy centrali
  75. Esercizi avanzati analisi

 
 

Tutte le cartelle di Analisi Matematica

Leggi...

  1. Prerequisiti di Analisi
    1. Ripasso algebra biennio liceo
    2. Ripasso geometria analitica
    3. Ripasso goniometria e trigonometria
    4. Errori tipici da evitare
    5. Insiemi numerici N,Z,Q,R
    6. Funzioni elementari
    7. Logica elementare
    8. Insiemi
  2. Successioni
    1. Teoria sulle Successioni
    2. Estremo superiore e inferiore
    3. Limiti base
    4. Forme indeterminate
    5. Limiti notevoli
    6. Esercizi misti Successioni
    7. Successioni per ricorrenza
  3. Funzioni
    1. Teoria sulle funzioni
    2. Verifica del limite in funzioni
    3. Limite base in funzioni
    4. Forme indeterminate in funzioni
    5. Limiti notevoli in funzioni
    6. Calcolo asintoti
    7. Studio di funzione senza derivate
    8. Dominio di una funzione
    9. Esercizi misti Funzioni
    10. Esercizi misti sui Limiti
  4. Funzioni continue-lipschitziane-holderiane
    1. Teoria sulle Funzioni continue-lipschitziane-holderiane
    2. Continuità delle funzioni
    3. Continuità uniforme
    4. Teorema degli zeri
    5. Esercizi sul teorema di Weierstrass senza l’uso delle derivate
  5. Calcolo differenziale
    1. Derivate
    2. Calcolo delle derivate
    3. Retta tangente nel calcolo differenziale
    4. Punti di non derivabilità nel calcolo differenziale
    5. Esercizi sul teorema di Weierstrass con l’uso delle derivate
    6. Studio di funzione completo nel calcolo differenziale
    7. Esercizi teorici nel calcolo differenziale
    8. Metodo di bisezione
    9. Metodo di Newton
  6. Teoremi del calcolo differenziale
    1. Teoria sui Teoremi del calcolo differenziale
    2. Teorema di Rolle
    3. Teorema di Lagrange
    4. Teorema di Cauchy
    5. Teorema di De L’Hôpital
  7. Calcolo integrale
    1. Integrale di Riemann
    2. Integrali immediati
    3. Integrale di funzione composta
    4. Integrali per sostituzione
    5. Integrali per parti
    6. Integrali di funzione razionale
    7. Calcolo delle aree
    8. Metodo dei rettangoli e dei trapezi
    9. Esercizi Misti Integrali Indefiniti
    10. Esercizi Misti Integrali Definiti
  8. Integrali impropri
    1. Teoria Integrali impropri
    2. Carattere di un integrale improprio
    3. Calcolo di un integrale improprio
  9. Espansione di Taylor
    1. Teoria Espansione di Taylor
    2. Limiti di funzione con Taylor
    3. Limiti di successione con Taylor
    4. Stime del resto
  10. Funzioni integrali (Approfondimento)
    1. Teoria Funzioni integrali (Approfondimento)
    2. Studio di funzione integrale
    3. Limiti con Taylor e De L’Hôpital
    4. Derivazione di integrali parametrici (Tecnica di Feynmann)
  11. Numeri Complessi
    1. Teoria Numeri complessi
    2. Espressioni con i numeri complessi
    3. Radice di un numero complesso
    4. Equazioni con i numeri complessi
    5. Disequazioni con i numeri complessi
    6. Esercizi misti Numeri complessi
  12. Serie numeriche
    1. Teoria Serie numeriche
    2. Esercizi Serie a termini positivi
    3. Esercizi Serie a termini di segno variabile
    4. Esercizi Serie geometriche e telescopiche
  13. Successioni di funzioni
    1. Teoria Successioni di funzioni
    2. Esercizi Successioni di funzioni
  14. Serie di funzioni
    1. Teoria Serie di funzioni
    2. Esercizi Serie di funzioni
  15. Serie di potenze
    1. Teoria Serie di potenze
    2. Esercizi Serie di potenze
  16. Serie di Fourier
    1. Teoria Serie di Fourier
    2. Esercizi Serie di Fourier
  17. Trasformata di Fourier
    1. Teoria Trasformata di Fourier
    2. Esercizi Trasformata di Fourier
  18. Funzioni di più variabili
    1. Teoria Funzioni di più variabili
    2. Massimi e minimi liberi e vincolati
    3. Limiti in due variabili
    4. Integrali doppi
    5. Integrali tripli
    6. Integrali di linea di prima specie
    7. Integrali di linea di seconda specie
    8. Forme differenziali e campi vettoriali
    9. Teorema di Gauss-Green
    10. Integrali di superficie
    11. Flusso di un campo vettoriale
    12. Teorema di Stokes
    13. Teorema della divergenza
    14. Campi solenoidali
    15. Teorema del Dini
  19. Equazioni differenziali lineari e non lineari
    1. Teoria equazioni differenziali lineari e non lineari
    2. Equazioni differenziali lineari e non lineari del primo ordine omogenee
  20. Equazioni differenziali lineari
    1. Del primo ordine non omogenee
    2. Di ordine superiore al primo,a coefficienti costanti,omogenee
    3. Di ordine superiore al primo,a coefficienti costanti,non omogenee
    4. Di Eulero,di Bernoulli,di Clairaut,di Lagrange e di Abel
    5. Non omogenee avente per omogenea associata un’equazione di Eulero
    6. Sistemi di EDO
  21. Equazioni differenziali non lineari
    1. A variabili separabiliO
    2. A secondo membro omogeneo
    3. Del tipo y’=y(ax+by+c)
    4. Del tipo y’=y(ax+by+c)/(a’x+b’y+c’)
    5. Equazioni differenziali esatte
    6. Mancanti delle variabili x e y
    7. Cenni sullo studio di un’assegnata equazione differenziale non lineare
    8. Di Riccati
    9. Cambi di variabile: simmetrie di Lie
  22. Analisi complessa
    1. Fondamenti
    2. Funzioni olomorfe
    3. Integrale di Cauchy e applicazioni
    4. Teorema della curva di Jordan e teorema fondamentale dell’Algebra
    5. Teorema di inversione di Lagrange
    6. Teorema dei Residui
    7. Funzioni meromorfe
    8. Prodotti infiniti e prodotti di Weierstrass
    9. Continuazione analitica e topologia
    10. Teoremi di rigidità di funzioni olomorfe
    11. Trasformata di Mellin
  23. Equazioni alle derivate parziali
    1. Equazioni del primo ordine
    2. Equazioni del secondo ordine lineari
    3. Equazioni non-lineari
    4. Sistemi di PDE
  24. Funzioni speciali
    1. Funzione Gamma di Eulero
    2. Funzioni Beta,Digamma,Trigamma
    3. Integrali ellittici
    4. Funzioni di Bessel
    5. Funzione zeta di Riemann e funzioni L di Dirichlet
    6. Funzione polilogaritmo
    7. Funzioni ipergeometriche
  25. Analisi funzionale
    1. Misura e integrale di Lebesgue
    2. Spazi Lp,teoremi di completezza e compattezza
    3. Spazi di Hilbert,serie e trasformata di Fourier
    4. Teoria e pratica dei polinomi ortogonali
    5. Spazi di Sobolev
  26. Complementi
    1. Curiosità e approfondimenti
    2. Compiti di analisi
    3. Esercizi avanzati analisi
  27. Funzioni Convesse

 
 

Tutti gli esercizi di geometria

In questa sezione vengono raccolti molti altri esercizi che coprono tutti gli argomenti di geometria proposti all’interno del sito con lo scopo di offrire al lettore la possibilità di approfondire e rinforzare le proprie competenze inerenti a tali argomenti.

Strutture algebriche.





 
 

Risorse didattiche aggiuntive per approfondire la matematica

Leggi...

  • Math Stack Exchange – Parte della rete Stack Exchange, questo sito è un forum di domande e risposte specificamente dedicato alla matematica. È una delle piattaforme più popolari per discutere e risolvere problemi matematici di vario livello, dall’elementare all’avanzato.
  • Art of Problem Solving (AoPS) – Questo sito è molto noto tra gli studenti di matematica di livello avanzato e i partecipanti a competizioni matematiche. Offre forum, corsi online, e risorse educative su una vasta gamma di argomenti.
  • MathOverflow – Questo sito è destinato a matematici professionisti e ricercatori. È una piattaforma per domande di ricerca avanzata in matematica. È strettamente legato a Math Stack Exchange ma è orientato a un pubblico con una formazione più avanzata.
  • PlanetMath – Una comunità collaborativa di matematici che crea e cura articoli enciclopedici e altre risorse di matematica. È simile a Wikipedia, ma focalizzata esclusivamente sulla matematica.
  • Wolfram MathWorld – Una delle risorse online più complete per la matematica. Contiene migliaia di articoli su argomenti di matematica, creati e curati da esperti. Sebbene non sia un forum, è una risorsa eccellente per la teoria matematica.
  • The Math Forum – Un sito storico che offre un’ampia gamma di risorse, inclusi forum di discussione, articoli e risorse educative. Sebbene alcune parti del sito siano state integrate con altri servizi, come NCTM, rimane una risorsa preziosa per la comunità educativa.
  • Stack Overflow (sezione matematica) – Sebbene Stack Overflow sia principalmente noto per la programmazione, ci sono anche discussioni rilevanti di matematica applicata, specialmente nel contesto della scienza dei dati, statistica, e algoritmi.
  • Reddit (r/Math) – Un subreddit popolare dove si possono trovare discussioni su una vasta gamma di argomenti matematici. È meno formale rispetto ai siti di domande e risposte come Math Stack Exchange, ma ha una comunità attiva e molte discussioni interessanti.
  • Brilliant.org – Offre corsi interattivi e problemi di matematica e scienza. È particolarmente utile per chi vuole allenare le proprie capacità di problem solving in matematica.
  • Khan Academy – Una risorsa educativa globale con lezioni video, esercizi interattivi e articoli su una vasta gamma di argomenti di matematica, dalla scuola elementare all’università.






Document









Document



error: Il contenuto è protetto!!